Consider the graph of the exponential function, y =3(2)^x. The x- intercept of the graph is

Answers

Answer 1

Answer:

the x intercept is 2

Step-by-step explanation:

I just went over this unit

Answer 2

Answer:

2

Step-by-step explanation:


Related Questions

What is the area of the triangle

Answers

Answer:

A. 6 inchesssssssss

Answer:

6

Step-by-step explanation:

A teacher of statistics wants to know if a new teaching methodology that includes IT is efficient in terms of increased average score. He took a class with old methodology and a class with new methodology for samples and gave a same test. Open the file by clicking the file name above. Once you open the file and run Excel, you need not open it again. What is Ha? Find it from Excel output that you generate.
a) 0.62.
b) 0.5.
c) 0.31.
d) -0.5.

Answers

Answer:

The answer is 0.31

Step-by-step explanation:

Old Method                                       New Method                          .

Mean                      73.5625                  Mean            75.70588

Standard Error        3.143736             Standard Error  2.923994

Median                    72                          Median            75

Mode                       90                          Mode               64

Standard deviation  12.57494           Standard deviation 12.05594

Sample Variance     158.1292            Sample Variance     145.3456

Kurtosis                     -1.14544              Kurtosis                   -0.76646

Skewness                 0.171025             Skewness                0.091008

Range                       39                        Range                      41

Minimum                   55                        Minimum                 56

Maximum                  94                        Maximum                 97

Sum                          1177                        Sum                        1287

Count                         16                         Count                      17

State the hypotheses. The first step is to state the null hypothesis and an alternative hypothesis.

Null hypothesis: μNew< μOld

Alternative hypothesis: μNew > μOld

Note that these hypotheses constitute a one-tailed test. The null hypothesis will be rejected if the mean difference between sample means is too small.

Formulate an analysis plan. For this analysis, the significance level is 0.05. Using sample data, we will conduct a two-sample t-test of the null hypothesis.

Analyze sample data. Using sample data, we compute the standard error (SE), degrees of freedom (DF), and the t statistic test statistic (t).

[tex]SE=\sqrt{\frac{S_1^2}{n_1} +\frac{S_2^2}{n_2} } \\\\SE=4.29[/tex]

DF = 31

[tex]t = \frac{(x_1-x_2)-d}{SE} \\\\t = - 0.4997[/tex]

where s1 is the standard deviation of sample 1, s2 is the standard deviation of sample 2, n1 is the size of sample 1, n2 is the size of sample 2, x1 is the mean of sample 1, x2 is the mean of sample 2, d is the hypothesized difference between population means, and SE is the standard error.

The observed difference in sample means produced a t statistic of - 0.499. We use the t Distribution Calculator to find P(t < - 0.499) = 0.311

Therefore, the P-value in this analysis is 0.311.

Interpret results. Since the P-value (0.311) is greater than the significance level (0.05), we cannot reject the null hypothesis.

From the above test we do have sufficient evidence in the favor of the claim that new method is efficient than the old method.

Simplify the expression below.
14a8y3 - 7 Ay5 + 28a12y2
7aty
A.
OB.
2a²y3 - ay5 + 4a3y2
2a4y? - JA + 428 y
2a4y3 – 5 + 428 y?
D. 2012,4 - 2876 +4215,3
C.

Answers

Answer:

14a8y3 - 7 Ay5 + 28a12y2-  7ay2 • (4a11 + 2a7y - y3)

Step-by-step explanation:

Equation at the end of step  1  :

 (((14•(a8))•(y3))-(7a•(y5)))+((22•7a12)•y2)

Step  2  :

Equation at the end of step  2  :

 (((14•(a8))•(y3))-7ay5)+(22•7a12y2)

Step  3  :

Equation at the end of step  3  :

 (((2•7a8) • y3) -  7ay5) +  (22•7a12y2)

Pull out like factors

Answer:   7ay2 • (4a11 + 2a7y - y3)

Hope this helps.

Solve 20x = 10 for x. A. x = 1/2 B. x = 1.5 C. x = 2 D. x = 10

Answers

Answer:

A. 1/2

Step-by-step explanation:

20x=10

Divide 20 on both sides of the equation to get x by itself

20x=10

___.  __

20.     20

x =1/2

Answer:

A) x= 1/2

Step-by-step explanation:

20x= 10 we then divide 10 by 20 to get x= 10/20 or if we simplify x= 1/2. Thus answer choice A) is correct!

Find the mean, median, mode and range for each set of data. Calculator usage is encouraged!
1. 23, 87, 19, 34, 37, 87, 81, 5, 14, 100, 26 Please help thank you!

Answers

Answer:

mean: 46.63636

median: 34

mode: 87

range:95

How To:

Step 1 : To find Mean

    Average = ( 1 + 5 + 5 + 7 + 8 + 10 ) / 6

             =36 / 6

    Mean = 6

Step 2 : To find Median

   Middle value = ( 5 + 7 ) / 2

               = 12 / 2

    Median = 6

Step 3 : To find Mode

   Mode = 5 (The number with more repetition, here 5 is repeated two times)

Step 4 : To find Range

    Range = Largest number - Smallest number

       = 10-1

       = 9

Range = 9

Answer:

Mean: 46.6

Mode: 87

Median: 34

Range: 95

Step-by-step explanation:

Mean: (finding the average)

Median: (the middle number of the data set)

Mode: (the most number repeated from the data set)

Range: (is the difference between the highest value and the lowest value)

first arrange the following data set.

23, 87, 19, 34, 37, 87, 81, 5, 14, 100, 26

so:

5, 14, 19, 23, 26, 34, 37, 81, 87, 87, 100

Lets us first find the mean by adding up all the numbers and dividing it by the amount of numbers in the data set.

Mean: 5 + 14 + 19 + 23 + 26 + 34 + 37 + 81 + 87 + 87 + 100 = 513/11 = 46.6

Mode: 87

Median: 34

Range: 100 - 5 = 95

Which expression is equivalent to m n + z?
n m + n
z + m z
m z + n
z + n m

Answers

Answer:

z + n m

Step-by-step explanation:

These expressions are equivalent because the commutative property of addition, which states that when adding two terms, the order doesn't matter.

If this answer is correct, please make me Brainliest!

Answer:

It's "c"

Step-by-step explanation:

i just did this on edge

To generate leads for new business, Gustin Investment Services offers free financial planning seminars at major hotels in Southwest Florida. Gustin conducts seminars for groups of 25 individuals. Each seminar costs Gustin $3,500, and the commission for each new account opened is $5,000. Gustin estimates that for each individual attending the seminar, there is a 0.01 probability that he/she will open a new account.

(a) Determine the equation for computing Gustin's profit per seminar, given values of the relevant parameters. Profit = (New Accounts Opened × ) –

(b) What type of random variable is the number of new accounts opened? (Hint: Review Appendix 11.1 for descriptions of various types of probability distributions.)

(c) Choose the appropriate spreadsheet simulation model to analyze the profitability of Gustin's seminars. (I) (II) (III) (IV) Would you recommend that Gustin continue running the seminars?

(d) How many attendees (in a multiple of five, i.e., 25, 30, 35, . . .) does Gustin need before a seminar's average expected profit is greater than zero?

Answers

Answer:

a) profit = (new account opened x 5000) -3500

b) Opening account is binomial distribution with n =25 and p = 0.01

c) Probability of loss is 0.77781 --I don't recommend the company that it running the seminar

d) n ≅ 71

Step-by-step explanation:

See attached image

Equations
What is the solution of the system of linear equations?
-3x + 4y = -18
2x - y = 7

(-2,-3)
(-2,3)
(2, -3)
(2, 3)​

Answers

Answer:

Step-by-step explanation:

-3x + 4y = -18

8x - 4y =   28

5x = 10

x = 2

4 - y = 7

-y = 3

y = -3

(2, -3)

The solution of the system of linear equations given is (2,-3), the correct option is C.

What is System of Linear Equation?

The system of linear equation is set of equations which have a common solution.

The equations are

-3x+4y = -18

2x-y =7

The linear equations can be solved using substitution method

y = 2x -7 from equation 2 will be substituted in equation 1

-3x +4 ( 2x -7) = -18

-3x +8x -28 = -18

5x = 10

x = 2

y = 2 * 2 -7 = -3

To know more about System of Linear Equations

https://brainly.com/question/12895249

#SPJ2

What is the number of possible permutations of 5 objects taken 2 at a time? A. 10 B. 20 C. 60 D. 120

Answers

Answer:

B. 20

Step-by-step explanation:

5P2 is equal to 20 using the permutation formula.

OMG THIS QUESTION IS SO HARD WILL RATE IF U GET IT

Answers

Answer:

19.5 in²

Step-by-step explanation:

Area of rhombus tile = side × height

Area = 3 × 6.5

Area = 19.5 in²

A rhombus, like any parallelogram, has area equal to base times height,

that's 3×6.5 = 19.5 square inches

Answer: 19.5

To the nearest tenth of a cubic centimeter, what is the volume of
the sphere if r = 17 cm?

Answers

20579.53 cubic centimeters.

Answer:

V≈20579.53

Step-by-step explanation:

[tex]V=\frac{4}{3} \pi r^3[/tex]

how many tenths are in 4600​

Answers

Answer:

4600 tenths as a Fraction

Since 4600 tenths is 4600 over ten, 4600 tenths as a Fraction is 4600/10.

4600 tenths as a Decimal

If you divide 4600 by ten you get 4600 tenths as a decimal which is 460.00.

4600 tenths as a Percent

To get 4600 tenths as a Percent, you multiply the decimal with 100 to get the answer of 46000 percent.

4600 tenths of a dollar

First we divide a dollar into ten parts where each part is 10 cents. Then we multiply 10 cents with 4600 and get 46000 cents or 460 dollars and 0 cents.

Step-by-step explanation:

Hope this helped!

Stay safe!!!

Answer:

Step-by-step explanation:

To answer this, multiply 4600 by 10:  46000.  There are 46000 tenths in 4600.

Which number line correctly shows 0.8 + 0.3?

Answers

Answer:

the second answer

Step-by-step explanation:

cause 0+0.8 is 0.8 and 0.8+0.3 is 1.1

Answer:

A

Step-by-step explanation:

Figure A is translated 3 units right and 2 units up. The translated figure is labeled figure B. Figure B is reflected over the x-axis. The reflected figure is labeled figure C. Which best explains why figure A is congruent to figure C? On a coordinate plane, triangle A has points (1, negative 2), (3, negative 2), (3, negative 5). Triangle B has points (4, 0), (6, 0), (6, negative 3). Triangle C has points (4, 0), (6, 0), (6, 3). A Is congruent to B and B Is congruent to C A Is congruent to A, B Is congruent to B, C Is congruent to C Each triangle is a right triangle. Each triangle is an isosceles triangle.

Answers

Answer:

A Is congruent to B and B Is congruent to C

Step-by-step explanation:

Let's look at the answer choices:

A: "A Is congruent to B and B Is congruent to C"

Well, clearly, if A ≅ B and B ≅ C, then by the transitive property, we can say that A ≅ C. So, A is very likely correct.

B: "A Is congruent to A, B Is congruent to B, C Is congruent to C"

Just because A is congruent to itself (and same with B and C) doesn't necessarily mean that they're congruent to each other. So, B is wrong.

C: "Each triangle is a right triangle."

Again, there are so many right triangles out there with different dimensions. For example, there are some with sides 3, 4, and 5, and others with sides 5, 12, and 13. They are not congruent, however. So, rule out C.

D: "Each triangle is an isosceles triangle."

This is just like choice C since there are so many variations of isosceles triangles. So D is wrong.

The answer is thus A.

Answer:

First one:

A Is congruent to B and B Is congruent to C

Step-by-step explanation:

Since B is obtained by translating A, it has the same measure angles and sides as A, hence B is congruent to A

C is obtained by reflecting B, which doesn't alter the measure of sides and angles, so C is congruent to B

Therefore by transition, C is congruent to A

Suppose you had to
guess on a four-choice
multiple-choice test and
were given four questions.
Find the binomial
probability distribution.
( + ) ℎ =
4 = 0.25

Answers

Answer:

For 0 correct answer [tex]^4c_0p^0q^{4-0}[/tex]

For 1 correct answer [tex]^4c_1p^1q^{4-1}[/tex]

For 2 correct answer [tex]^4c_2p^0q^{4-2}[/tex]

For 3 correct answer [tex]^4c_3p^1q^{4-3}[/tex]

For 4 correct answer [tex]^4c_4p^1q^{4-4}[/tex]

Step-by-step explanation:

It is given that there are 4 questions n = 4

Number of choices is 4

So probability of getting correct answer [tex]=\frac{1}{4}[/tex]

Probability of getting incorrect answer [tex]=1-\frac{1}{4}=\frac{3}{4}[/tex]

Probability distribution is given by [tex]^nc_rp^rq^{n-r}[/tex]

Therefore probability distribution of 0 correct answer

[tex]^4c_0p^0q^{4-0}[/tex]

Therefore probability distribution of 1 correct answer

[tex]^4c_1p^1q^{4-1}[/tex]

Therefore probability distribution of 2 correct answer

[tex]^4c_2p^0q^{4-2}[/tex]

Therefore probability distribution of 3 correct answer.

[tex]^4c_3p^1q^{4-3}[/tex]

Therefore probability distribution of 4 correct answer.

[tex]^4c_4p^1q^{4-4}[/tex]

Suppose the round-trip airfare between Philadelphia and Los Angeles a month before the departure date follows the normal probability distribution with a mean of $387.20 and a standard deviation of $68.50. What is the probability that a randomly selected airfare between these two cities will be between $325 and $425?

Answers

Answer:

52.74% probability that a randomly selected airfare between these two cities will be between $325 and $425

Step-by-step explanation:

Problems of normally distributed samples are solved using the z-score formula.

In a set with mean [tex]\mu[/tex] and standard deviation [tex]\sigma[/tex], the zscore of a measure X is given by:

[tex]Z = \frac{X - \mu}{\sigma}[/tex]

The Z-score measures how many standard deviations the measure is from the mean. After finding the Z-score, we look at the z-score table and find the p-value associated with this z-score. This p-value is the probability that the value of the measure is smaller than X, that is, the percentile of X. Subtracting 1 by the pvalue, we get the probability that the value of the measure is greater than X.

In this question, we have that:

[tex]\mu = 387.20, \sigma = 68.50[/tex]

What is the probability that a randomly selected airfare between these two cities will be between $325 and $425?

This is the pvalue of Z when X = 425 subtracted by the pvalue of Z when X = 325. So

X = 425

[tex]Z = \frac{X - \mu}{\sigma}[/tex]

[tex]Z = \frac{425 - 387.20}{68.50}[/tex]

[tex]Z = 0.55[/tex]

[tex]Z = 0.55[/tex] has a pvalue of 0.7088

X = 325

[tex]Z = \frac{X - \mu}{\sigma}[/tex]

[tex]Z = \frac{325 - 387.20}{68.50}[/tex]

[tex]Z = -0.91[/tex]

[tex]Z = -0.91[/tex] has a pvalue of 0.1814

0.7088 - 0.1814 = 0.5274

52.74% probability that a randomly selected airfare between these two cities will be between $325 and $425

Junior bought a bag of mixed fruit snacks. The flavors in the bag are 4 strawberry, 3 cherry, and 5 grape. If he chooses one fruit snack at random, what it the probability of the first one being grape?

Answers

Answer:I believe it would be 5/12

Step-by-step explanation:

You add all of them up then since it's 5 grapes and in total there is 12 fruit snacks. It should be 5 grapes of 12 fruit snacks in the bag.

In ΔXYZ, the measure of ∠Z=90°, the measure of ∠X=57°, and XY = 8 feet. Find the length of YZ to the nearest tenth of a foot.

Answers

Answer:

21

Step-by-step explanation:

Answer:

6.7

Step-by-step explanation:


You roll a six-sided number cube (die). What is the BEST answer for the probability that the number rolled is between 1 and 6, inclusive?
A) certain
B) unlikely
C) impossible
D) very likely

Answers

Answer: It is A certain.

Step-by-step explanation:

Because all the numbers on a six-sided cube is between 1 and 6 so it is certain or 100/100 that the number will land on a number between 1 and 6.

Drag each tile to the correct box. Not all tiles will be used.
Arrange the equations in the correct sequence to find the inverse of f(x) = y = 3x / 8 + x​

Answers

Answer:

Inverse of f(x)

               [tex]f^{l} (x) = \frac{8 x}{3-x}[/tex]

Step-by-step explanation:

Explanation:-

Step(i):-

Given the function

                        [tex]f(x) = \frac{3 x}{8+x}[/tex]

Given function is one-one and onto function

Hence f(x) is bijection function

                   [tex]y = f(x) = \frac{3 x}{8+x}[/tex]

now cross multiplication, we get

            ( 8+x)y = 3 x

             8 y + x y = 3 x

             8 y = 3 x - x y

taking Common 'x' we get

            x (3 - y) = 8 y

                   [tex]x = \frac{8 y}{3-y}[/tex]

Step(ii):-

The inverse function

                 [tex]x = \frac{8 y}{3-y} = f^{l}(y)[/tex]

The inverse function of x

                  [tex]f^{l}(x) = \frac{8 x}{3-x}[/tex]

Final answer:-

Inverse of f(x)

               [tex]f^{l} (x) = \frac{8 x}{3-x}[/tex]

Plz help ..............!!!!!

Answers

Answer:

1.8

is the median

Answer: 1.8

Step-by-step explanation: 1.8 is the median

Two samples each of size 20 are taken from independent populations assumed to be normally distributed with equal variances. The first sample has a mean of 43.5 and standard deviation of 4.1 while the second sample has a mean of 40.1 and standard deviation of 3.2. A researcher would like to test if there is a difference between the population means at the 0.05 significance level. What can the researcher conclude?

Answers

Answer:

[tex]t=\frac{(43.5 -40.1)-(0)}{3.678\sqrt{\frac{1}{20}+\frac{1}{20}}}=2.923[/tex]

The degrees of freedom are

[tex]df=20+20-2=38[/tex]

And the p value is given by:

[tex]p_v =2*P(t_{38}>2.923) =0.0058[/tex]

Since the p value for this cae is lower than the significance level of 0.05 we have enough evidence to reject the null hypothesis and we can conclude that the true means for this case are significantly different

Step-by-step explanation:

When we have two independent samples from two normal distributions with equal variances we are assuming that  

[tex]\sigma^2_1 =\sigma^2_2 =\sigma^2[/tex]

And the statistic is given by this formula:

[tex]t=\frac{(\bar X_1 -\bar X_2)-(\mu_{1}-\mu_2)}{S_p\sqrt{\frac{1}{n_1}+\frac{1}{n_2}}}[/tex]

Where t follows a t distribution with [tex]n_1+n_2 -2[/tex] degrees of freedom and the pooled variance [tex]S^2_p[/tex] is given by this formula:

[tex]S^2_p =\frac{(n_1-1)S^2_1 +(n_2 -1)S^2_2}{n_1 +n_2 -2}[/tex]

The system of hypothesis on this case are:

Null hypothesis: [tex]\mu_1 = \mu_2[/tex]

Alternative hypothesis: [tex]\mu_1 \neq \mu_2[/tex]

We have the following data given:

[tex]n_1 =20[/tex] represent the sample size for group 1

[tex]n_2 =20[/tex] represent the sample size for group 2

[tex]\bar X_1 =43.5[/tex] represent the sample mean for the group 1

[tex]\bar X_2 =40.1[/tex] represent the sample mean for the group 2

[tex]s_1=4.1[/tex] represent the sample standard deviation for group 1

[tex]s_2=3.2[/tex] represent the sample standard deviation for group 2

First we can begin finding the pooled variance:

[tex]\S^2_p =\frac{(20-1)(4.1)^2 +(20 -1)(3.2)^2}{20 +20 -2}=13.525[/tex]

And the deviation would be just the square root of the variance:

[tex]S_p=3.678[/tex]

The statistic is givne by:

[tex]t=\frac{(43.5 -40.1)-(0)}{3.678\sqrt{\frac{1}{20}+\frac{1}{20}}}=2.923[/tex]

The degrees of freedom are

[tex]df=20+20-2=38[/tex]

And the p value is given by:

[tex]p_v =2*P(t_{38}>2.923) =0.0058[/tex]

Since the p value for this cae is lower than the significance level of 0.05 we have enough evidence to reject the null hypothesis and we can conclude that the true means for this case are significantly different

Using the t-distribution, as we have the standard deviation for the sample, it is found that the researcher can conclude that there is a difference between the population means at the 0.05 significance level.

What are the hypothesis tested?

At the null hypothesis, it is tested if there is no difference, that is:

[tex]H_0: \mu_1 - \mu_2 = 0[/tex]

At the alternative hypothesis, it is tested if there is a difference, that is:

[tex]H_a: \mu_1 - \mu_2 \neq 0[/tex]

What is the mean and the standard error of the distribution of differences?

For each sample, we have that they are given by

[tex]\mu_1 = 43.5, s_1 = \frac{4.1}{\sqrt{20}} = 0.9168[/tex]

[tex]\mu_2 = 40.2, s_2 = \frac{3.2}{\sqrt{20}} = 0.7155[/tex]

Hence, for the distribution of differences, the mean and the standard error are given by:

[tex]\overline{x} = \mu_1 - \mu_2 = 43.5 - 40.2 = 3.3[/tex]

[tex]s = \sqrt{s_1^2 + s_2^2} = \sqrt{0.9168^2 + 0.7155^2} = 1.163[/tex]

What is the test statistic?

It is given by:

[tex]t = \frac{\overline{x} - \mu}{s}[/tex]

In which [tex]\mu = 0[/tex] is the value tested at the null hypothesis, hence:

[tex]t = \frac{\overline{x} - \mu}{s}[/tex]

[tex]t = \frac{3.3 - 0}{1.163}[/tex]

[tex]t = 2.84[/tex]

What is the decision?

Considering a two-tailed test, as we are testing if the mean is different of a value, with a significance level of 0.05 and 20 + 20 - 2 = 38 df, the critical value is of [tex]|z^{\ast}| = 2.0244[/tex].

Since the absolute value of the test statistic is greater than the critical value, it is found that the researcher can conclude that there is a difference between the population means at the 0.05 significance level.

More can be learned about the t-distribution at https://brainly.com/question/16313918

4 ÷ 1/5 = 20 because​

Answers

Step-by-step explanation:

BECAUUUUSE ;

[tex]4 \div \frac{1}{5} = 20 \\ \frac{4}{1} \div \frac{1}{5} = 20 \\ \frac{4}{1} \times \frac{5}{1} = 20 \\ \frac{20}{1} = 20[/tex]

Answer:Because the divide sign change to multiplication sign, and when this happens denominator in the right hand side will become numerator,while the formal numerator will become denominator.

Step-by-step explanation:

4 ➗ 1/5

4 x 5/1=20

Solve.
A European swallow flies about 12 meters in 1 second.
How many kilometers could it fly in 15 minutes?
It could fly |​

Answers

Answer:

It could fly 10,8 kilometers.

Step-by-step explanation:

The European swallow fly speed is 12 meters per second.

We have to calculate how many kilometers it could fly in 15 minutes.

This can be calculated using the equivalent factors for each of the units:

- 1 km is equivalent to 1,000 meters.

- 1 minute is equivalent to 60 seconds.

We know that the distance travelled by the fly is the product of the speed and the time, so we have:

[tex]D=v\cdot t\\\\\\D=12\,\dfrac{m}{s}\cdot15\, min\\\\\\D=12\,\dfrac{m}{s}\cdot(\dfrac{1\,km}{1,000\,m})\cdot15\, min\cdot (\dfrac{60\,s}{1\,min})\\\\\\D=\dfrac{12\cdot 15\cdot 60}{1,000}\,km=\dfrac{10,800}{1,000}\,km\\\\\\D=10,8 \,km[/tex]

Ramesh examined the pattern in the table. Powers of 7 Value 7 Superscript 4 2,401 7 Superscript 3 343 7 Superscript 2 49 7 Superscript 1 7 7 Superscript 0 1 7 Superscript negative 1 StartFraction 1 Over 7 EndFraction Ramesh says that based on the pattern 7 Superscript negative 5 = negative 16,807. Which statement explains whether Ramesh is correct? Ramesh is correct because 7 Superscript negative 5 is equivalent to Negative 7 times (negative 7) times (negative 7) times (negative 7) times (negative 7), which has the same value as Negative 16,807. Ramesh is correct because as the exponents decrease, the previous value is divided by 7, so 7 Superscript negative 5 = 1 divided by 7 divided by 7 divided by 7 divided by 7 divided by 7 = negative 16,807. Ramesh is not correct because 7 Superscript negative 5 is equivalent to StartFraction 1 Over 7 Superscript 5 EndFraction, which has the same value as StartFraction 1 Over 7 Superscript 4 EndFraction divided by 7 = StartFraction 1 Over 7 cubed EndFraction = StartFraction 1 Over 343 EndFraction. Ramesh is not correct because as the exponents decrease, the previous value is divided by 7, so 7 Superscript negative 5 = 1 divided by 7 divided by 7 divided by 7 divided by 7 divided by 7 = StartFraction 1 Over 16,807 EndFraction. NEED HELP NOW PLEASE I HAVE ONLY SEEN WRONG ANSWERS

Answers

Answer:

D

Step-by-step explanation:

Answer:

D.- Ramesh is not correct because as the exponents decrease, the previous value is divided by 7, so 7^-5 = 1 ÷ 7 ÷ 7 ÷ 7 ÷ 7 ÷ 7 = 1/16,807.

A footbridge has a span of 54 feet. A sign is
to be placed exactly halfway across the bridge. How far will the center of the sign be from each end of the bridge?

Answers

Answer:

27

Step-by-step explanation:

Because if it is halfway, that means

halfway=1/2

1/2=1/2 of 54

54/2 or 1/2 of 54=27

PLS MARK ME BRAINLIEST I NEED IT PLEASE

The center of the sign will be 27 feet apart from both ends of the bridge.

Given that,
A footbridge has a span of 54 feet. A sign is to be placed exactly halfway across the bridge. How far will the center of the sign be from each end of the bridge is to be determined.

What is arithmetic?

In mathematics, it deals with numbers of operations according to the statements. There are four major arithmetic operators, addition, subtraction, multiplication, and division,

Here,

Since the bridge is 54 feet long,
Now at the center of the bridge, a sign is placed,
So the distance of sign from both ends is equal to half of the total length of the bridge. i.e.
= 54 / 2
= 27

Thus, the center of the sign will be 27 feet apart from both ends of the bridge.

Learn more about arithmetic here:

brainly.com/question/14753192

#SPJ2



will mark the branliest to first one who answers

Answers

Answer:

3 1/4

Step-by-step explanation:

3/4 + (1/3 ÷1/6) - (-1/2)

Subtracting a negative is adding

3/4 + (1/3 ÷1/6) +1/2

Parentheses first

Copy dot flip

3/4 + (1/3 * 6/1) +1/2

3/4 + 2 + 1/2

Get a common denominator

3/4 + 2 + 2/4

2 + 5/4

2 + 4/4 +1/4

2+1 + 1/4

3 1/4

Im a parent for a 5th grader and don't remember this, plz help?
Again,
Josh wants to make 5 airplane propellers. he needs 18 centimeters of wood for each propeller. how many centimeters of wood will he use? How can I help him understand this problem.

Answers

Answer:

90 cm.

Step-by-step explanation:

One airplane propeller needs 18 centimetres of wood.

Josh wants to make 5 of them.

So, we would have to take the '18' centimetres of wood and multiply by 5 to get the total for all 5 pieces.

[tex]\text{Number of Propellers} * 18 \text{ Centimetres}[/tex]

[tex]5 * 18 = 90[/tex]

Josh should need 90 centimetres of wood total to make 5 airplane propellers.

So since josh wants to make 5 airplane propellers and he would need 18 centimeters of wood for each propeller. He would need 90cm of wood. When I was younger I would just add. For this problem I would just do this. 18+18+18+18+18=90. I would suggest doing this if your child isn’t very good a multiplication. I’m sorry if this is wrong or something it’s been 7 years since I have been to the 5th grade.

Answer=90

What is the vertex of the graph of the function f(x) = x2 + 8x - 2 ?
(-4, 18)
(0, -2)
(-8, -2)
(-4, -18)

Answers

Answer: (-4,-18)

Step-by-step explanation: If you use desmos you can graph the equation to find the vertex.

The vertex of the graph of function f(x) =  x² + 8x - 2 is (-4, -18)

What is the vertex of the graph of a quadratic function?

In a quadratic function, the vertex of the graph refers to the highest or lowest possible outcome of the function. In a graph, the vertex is the highest or lowest point on the parabola,

Given that:

f(x) = x² + 8x - 2

where;

a = 1b = 8c = - 2

By using the vertex formula to find the x-value;

[tex]\mathbf{x = \dfrac{-b}{2a}}[/tex]

[tex]\mathbf{x = \dfrac{-8}{2(1)}}[/tex]

x = -4

So,

y = (-4)² + 8(-4) - 2

y = 16 -32 -2

y = -18

Therefore, the vertex of the graph of function f(x) =  x² + 8x - 2 is (-4, -18)

Learn more about the vertex of the graph function here:

https://brainly.com/question/1305555

#SPJ2

2x + 3y = 12
Complete the missing value in the solution to the equation.
,8)

Answers

Answer: -6

Step by step:
2x + 3(8) = 12
2x + 24 = 12
2x = -12
x = -6
Other Questions
Calculate the molar mass of ammonium chloride Reemplace el complemento directo por el pronombre de objecto directo.Los estudiantes entregan las tareas.Los estudiantesentregan.0A. losB. lasC. laD. lo -3x+1/2=3/8Whats x? pls someone help step by step What is the answer? Plz need help Which part of the Roman Republic did Caesar Augustus restore? Which statement is an infrence about the odyssey? Kendra was removed from the voting committee for Homecoming King because oneof the nominees was her boyfriend. Why was she removed?biasstereotypeOOOmanipulation of informationvested interest 13th term of 2, 10, 18, 26 If cos A = 12/13 , and A is in the fourth quadrant, find the exact value of sin2A. A steam engine takes in superheated steam at 270 C and discharges condensed steam from its cylinder at 50 C. The engine has an efficiency of 30%, and taken in 50 kJ from the hot steam per cycle. If a Carnot engine takes in the same amount of heat per cycle and operates at these temperatures, the work it can turn into is most likely to be:a) 15 kJ. b) 20 kJ. c) 10 kJ. d) 50 kJ. There are four countries in the region of Central Asia.Please select the best answer from the choices providedTF In which year did BEACH volleyball first become an olympic game IS IT right to the left a b c d A typical cell phone uses wavelength of 0.36meters. How much energy does this wave have? damage to the skin and underlying tissues caused by prolonged exposure to extreme cold is called ________ discuss methods that individuals, groups and/ or the government have used since 1950 to deal with the inequality faced by african americans. help pls What does the questions how much? and how many? have in common? 5(x-10)=30-15xOx=1Ox=4Ox=5Ox=8 How big was the Inca empire? List at least three events that have occurred since the begin-ning of the school year that you think will be included infuture history books. Support your answers.